How to take every third element in a series?












5












$begingroup$


$sum_{n=1}^infty frac{1}{n^2} = frac{1}{1^2} + frac{1}{2^2} + frac{1}{3^2} + cdots = 1.644934$ or $frac{pi^2}{6}$



What if we take every 3rd term and add them up?



A = $ frac{1}{3^2} + frac{1}{6^2} + frac{1}{9^2} + cdots = ??$



How to take every 3rd-1 term and add them up?



B = $ frac{1}{2^2} + frac{1}{5^2} + frac{1}{8^2} + cdots = ??$



How to take every 3rd-2 term and add them up?



C = $ frac{1}{1^2} + frac{1}{4^2} + frac{1}{7^2} + cdots = ??$



I am not sure how to adapt Eulers methods as he used the power series of sin for his arguments: https://en.wikipedia.org/wiki/Basel_problem










share|cite|improve this question











$endgroup$








  • 1




    $begingroup$
    The first sum you mentioned is $frac{1}{9}$ of the initial sum.
    $endgroup$
    – Peter
    Jan 3 at 16:26










  • $begingroup$
    The first sum can be rewritten as$$frac 19+frac 1{36}+cdots=frac 19left[1+frac 14+frac 19+cdotsright]color{blue}{=frac 19zeta(2)}$$
    $endgroup$
    – Frank W.
    Jan 3 at 16:30










  • $begingroup$
    If I may, but what is this problem for? When would you ever need to calculate each sum?
    $endgroup$
    – Frank W.
    Jan 3 at 16:31










  • $begingroup$
    @Peter that means that the other two sums must be different. as if they were the same the total would be $frac{3}{9}$ the initial sum.
    $endgroup$
    – Dale
    Jan 3 at 16:38












  • $begingroup$
    @Dale Peter said the first sum
    $endgroup$
    – Frank W.
    Jan 3 at 16:43
















5












$begingroup$


$sum_{n=1}^infty frac{1}{n^2} = frac{1}{1^2} + frac{1}{2^2} + frac{1}{3^2} + cdots = 1.644934$ or $frac{pi^2}{6}$



What if we take every 3rd term and add them up?



A = $ frac{1}{3^2} + frac{1}{6^2} + frac{1}{9^2} + cdots = ??$



How to take every 3rd-1 term and add them up?



B = $ frac{1}{2^2} + frac{1}{5^2} + frac{1}{8^2} + cdots = ??$



How to take every 3rd-2 term and add them up?



C = $ frac{1}{1^2} + frac{1}{4^2} + frac{1}{7^2} + cdots = ??$



I am not sure how to adapt Eulers methods as he used the power series of sin for his arguments: https://en.wikipedia.org/wiki/Basel_problem










share|cite|improve this question











$endgroup$








  • 1




    $begingroup$
    The first sum you mentioned is $frac{1}{9}$ of the initial sum.
    $endgroup$
    – Peter
    Jan 3 at 16:26










  • $begingroup$
    The first sum can be rewritten as$$frac 19+frac 1{36}+cdots=frac 19left[1+frac 14+frac 19+cdotsright]color{blue}{=frac 19zeta(2)}$$
    $endgroup$
    – Frank W.
    Jan 3 at 16:30










  • $begingroup$
    If I may, but what is this problem for? When would you ever need to calculate each sum?
    $endgroup$
    – Frank W.
    Jan 3 at 16:31










  • $begingroup$
    @Peter that means that the other two sums must be different. as if they were the same the total would be $frac{3}{9}$ the initial sum.
    $endgroup$
    – Dale
    Jan 3 at 16:38












  • $begingroup$
    @Dale Peter said the first sum
    $endgroup$
    – Frank W.
    Jan 3 at 16:43














5












5








5


1



$begingroup$


$sum_{n=1}^infty frac{1}{n^2} = frac{1}{1^2} + frac{1}{2^2} + frac{1}{3^2} + cdots = 1.644934$ or $frac{pi^2}{6}$



What if we take every 3rd term and add them up?



A = $ frac{1}{3^2} + frac{1}{6^2} + frac{1}{9^2} + cdots = ??$



How to take every 3rd-1 term and add them up?



B = $ frac{1}{2^2} + frac{1}{5^2} + frac{1}{8^2} + cdots = ??$



How to take every 3rd-2 term and add them up?



C = $ frac{1}{1^2} + frac{1}{4^2} + frac{1}{7^2} + cdots = ??$



I am not sure how to adapt Eulers methods as he used the power series of sin for his arguments: https://en.wikipedia.org/wiki/Basel_problem










share|cite|improve this question











$endgroup$




$sum_{n=1}^infty frac{1}{n^2} = frac{1}{1^2} + frac{1}{2^2} + frac{1}{3^2} + cdots = 1.644934$ or $frac{pi^2}{6}$



What if we take every 3rd term and add them up?



A = $ frac{1}{3^2} + frac{1}{6^2} + frac{1}{9^2} + cdots = ??$



How to take every 3rd-1 term and add them up?



B = $ frac{1}{2^2} + frac{1}{5^2} + frac{1}{8^2} + cdots = ??$



How to take every 3rd-2 term and add them up?



C = $ frac{1}{1^2} + frac{1}{4^2} + frac{1}{7^2} + cdots = ??$



I am not sure how to adapt Eulers methods as he used the power series of sin for his arguments: https://en.wikipedia.org/wiki/Basel_problem







power-series






share|cite|improve this question















share|cite|improve this question













share|cite|improve this question




share|cite|improve this question








edited Jan 13 at 19:36







Dale

















asked Jan 3 at 16:25









DaleDale

1,1801334




1,1801334








  • 1




    $begingroup$
    The first sum you mentioned is $frac{1}{9}$ of the initial sum.
    $endgroup$
    – Peter
    Jan 3 at 16:26










  • $begingroup$
    The first sum can be rewritten as$$frac 19+frac 1{36}+cdots=frac 19left[1+frac 14+frac 19+cdotsright]color{blue}{=frac 19zeta(2)}$$
    $endgroup$
    – Frank W.
    Jan 3 at 16:30










  • $begingroup$
    If I may, but what is this problem for? When would you ever need to calculate each sum?
    $endgroup$
    – Frank W.
    Jan 3 at 16:31










  • $begingroup$
    @Peter that means that the other two sums must be different. as if they were the same the total would be $frac{3}{9}$ the initial sum.
    $endgroup$
    – Dale
    Jan 3 at 16:38












  • $begingroup$
    @Dale Peter said the first sum
    $endgroup$
    – Frank W.
    Jan 3 at 16:43














  • 1




    $begingroup$
    The first sum you mentioned is $frac{1}{9}$ of the initial sum.
    $endgroup$
    – Peter
    Jan 3 at 16:26










  • $begingroup$
    The first sum can be rewritten as$$frac 19+frac 1{36}+cdots=frac 19left[1+frac 14+frac 19+cdotsright]color{blue}{=frac 19zeta(2)}$$
    $endgroup$
    – Frank W.
    Jan 3 at 16:30










  • $begingroup$
    If I may, but what is this problem for? When would you ever need to calculate each sum?
    $endgroup$
    – Frank W.
    Jan 3 at 16:31










  • $begingroup$
    @Peter that means that the other two sums must be different. as if they were the same the total would be $frac{3}{9}$ the initial sum.
    $endgroup$
    – Dale
    Jan 3 at 16:38












  • $begingroup$
    @Dale Peter said the first sum
    $endgroup$
    – Frank W.
    Jan 3 at 16:43








1




1




$begingroup$
The first sum you mentioned is $frac{1}{9}$ of the initial sum.
$endgroup$
– Peter
Jan 3 at 16:26




$begingroup$
The first sum you mentioned is $frac{1}{9}$ of the initial sum.
$endgroup$
– Peter
Jan 3 at 16:26












$begingroup$
The first sum can be rewritten as$$frac 19+frac 1{36}+cdots=frac 19left[1+frac 14+frac 19+cdotsright]color{blue}{=frac 19zeta(2)}$$
$endgroup$
– Frank W.
Jan 3 at 16:30




$begingroup$
The first sum can be rewritten as$$frac 19+frac 1{36}+cdots=frac 19left[1+frac 14+frac 19+cdotsright]color{blue}{=frac 19zeta(2)}$$
$endgroup$
– Frank W.
Jan 3 at 16:30












$begingroup$
If I may, but what is this problem for? When would you ever need to calculate each sum?
$endgroup$
– Frank W.
Jan 3 at 16:31




$begingroup$
If I may, but what is this problem for? When would you ever need to calculate each sum?
$endgroup$
– Frank W.
Jan 3 at 16:31












$begingroup$
@Peter that means that the other two sums must be different. as if they were the same the total would be $frac{3}{9}$ the initial sum.
$endgroup$
– Dale
Jan 3 at 16:38






$begingroup$
@Peter that means that the other two sums must be different. as if they were the same the total would be $frac{3}{9}$ the initial sum.
$endgroup$
– Dale
Jan 3 at 16:38














$begingroup$
@Dale Peter said the first sum
$endgroup$
– Frank W.
Jan 3 at 16:43




$begingroup$
@Dale Peter said the first sum
$endgroup$
– Frank W.
Jan 3 at 16:43










4 Answers
4






active

oldest

votes


















6












$begingroup$

Note that we have



$$psi'(z)=sum_{n=0}^infty frac{1}{(n+z)^2}$$



where $psi'(z)$ is the derivative of the digamma function. Hence, we can write



$$sum_{n=0}^infty frac{1}{(3n+1)^2}=frac19 psi'(1/3)$$



and



$$sum_{n=0}^infty frac{1}{(3n+2)^2}=frac19 psi'(2/3)$$



Interestingly, since we have



$$sum_{n=0}^infty left(frac1{(3n+3)^2}+frac1{(3n+2)^2}+frac1{(3n+1)^2}right)=frac{pi^2}{6}$$



we find that



$$psi'(1/3)+psi'(2/3) = 4pi^2/3$$






share|cite|improve this answer











$endgroup$













  • $begingroup$
    Why does one psi lack a '
    $endgroup$
    – Dale
    Jan 13 at 18:13










  • $begingroup$
    @dale That ommission is an obvious typographical error. I've edited accordingly.
    $endgroup$
    – Mark Viola
    Jan 13 at 18:59










  • $begingroup$
    Yes, at least one of us. Note that $sum_{n=0}^infty left(frac1{(3n+3)^2}+frac1{(3n+2)^2}+frac1{(3n+1)^2}right)=frac{pi^2}{6}$ and hence $$frac19(psi'(1/3)+psi'(2/3))=frac{pi^2}{6}-frac{pi^2}{54}=frac{8pi^2}{54}=frac19left(frac{4pi^2}{3}right)$$Now multiply by $9$.
    $endgroup$
    – Mark Viola
    Jan 13 at 19:29



















3












$begingroup$

As a complement to Mark's answer,



$$sum_{ngeq 0}frac{1}{(3n+1)^2}=-int_{0}^{1}sum_{ngeq 0} x^{3n}log(x),dx=int_{0}^{1}frac{-log x}{1-x^3},dx $$
(and similarly $sum_{ngeq 0}frac{1}{(3n+2)^2}$) can be expressed in terms of dilogarithms, since
$$ int_{0}^{1}frac{-log x}{1-a x}=frac{text{Li}_2(a)}{a} $$
for any $|a|leq 1$, with $text{Li}_2(a)=sum_{ngeq 1}frac{a^n}{n^2}$. This is equivalent to stating that $psi'left(frac{1}{3}right)$ and $psi'left(frac{2}{3}right)$ can be computed through the discrete Fourier transform. It is worth noticing that
$$text{Re},text{Li}_2(e^{itheta})=sum_{ngeq 1}frac{cos(ntheta)}{n^2} $$
is a continuous and piecewise-parabolic function, as the formal primitive of the sawtooth wave. On the contrary, $text{Im},text{Li}_2(e^{itheta})$ does not have a nice closed form, in general. Ref.: https://en.wikipedia.org/wiki/Spence%27s_function






share|cite|improve this answer









$endgroup$





















    1












    $begingroup$

    Polylogarithms



    A useful formula that can be applied here is
    $$
    frac13sum_{k=0}^2e^{2pi ijk/3}=[3mid j]tag1
    $$

    So
    $$
    begin{align}newcommand{Li}{operatorname{Li}}
    sum_{j=0}^inftyfrac1{(3j+1)^2}
    &=frac13sum_{k=0}^2sum_{j=1}^infty e^{2pi i(j-1)k/3}frac1{j^2}\
    &=frac13left(frac{pi^2}6+e^{-2pi i/3}Li_2left(e^{2pi i/3}right)+e^{2pi i/3}Li_2left(e^{-2pi i/3}right)right)tag2
    end{align}
    $$

    Mathematica gives $1.12173301393634378687$ using
    N[1/3(Pi^2/6 + Exp[-2Pi I/3]PolyLog[2,Exp[2Pi I/3]]+
    Exp[2Pi I/3] PolyLog[2,Exp[-2Pi I/3]]),20]





    Extended Harmonic Numbers



    Another approach is to use the Extended Harmonic Numbers.
    $$
    H(x)=sum_{k=1}^inftyleft(frac1k-frac1{k+x}right)tag3
    $$

    where
    $$
    H'(x)=sum_{k=1}^inftyfrac1{(k+x)^2}tag5
    $$

    Giving
    $$
    frac19H'!left(-frac23right)=sum_{k=0}^inftyfrac1{(3k+1)^2}tag6
    $$

    Mathematica gives $1.1217330139363437869$ using
    N[1/9HarmonicNumber'[-2/3],20]





    Euler-Maclaurin Sum Formula



    Although it does not give a closed form, the Euler-Maclaurin Sum Formula allows us to accelerate the computation of the sum.
    $$
    begin{align}
    sum_{k=0}^nfrac1{(3k+1)^2}
    &sim C-frac1{3(3n+1)}+frac1{2(3n+1)^2}-frac1{2(3n+1)^3}+frac9{10(3n+1)^5}\
    &-frac{81}{14(3n+1)^7}+frac{729}{10(3n+1)^9}-frac{32805}{22(3n+1)^{11}}tag7
    end{align}
    $$

    Using $n=100$ in $(7)$, we get
    $$
    sum_{k=0}^inftyfrac1{(3k+1)^2}=1.1217330139363437868657782tag8
    $$






    share|cite|improve this answer











    $endgroup$





















      -1












      $begingroup$

      KM101 deleted his hint... not sure why.



      $frac{1}{3^2}+frac{1}{6^2}+frac{1}{9^2}+dots=frac{1}{3^2 1^2}+frac{1}{3^2 2^2}+frac{1}{3^2 3^3}+dots=frac{1}{9}(frac{1}{1^2}+frac{1}{2^2}+frac{1}{3^2}+dots)= frac{pi^2}{54}$



      $frac{1}{2^2}+frac{1}{5^2}+frac{1}{8^2}+dots=frac{1}{2^2 1^2}+frac{1}{3^2?? 1^2}+frac{1}{3^2 ??1^2}$






      share|cite|improve this answer











      $endgroup$









      • 1




        $begingroup$
        How does this answer the OP's question?
        $endgroup$
        – Mark Viola
        Jan 3 at 20:24











      Your Answer





      StackExchange.ifUsing("editor", function () {
      return StackExchange.using("mathjaxEditing", function () {
      StackExchange.MarkdownEditor.creationCallbacks.add(function (editor, postfix) {
      StackExchange.mathjaxEditing.prepareWmdForMathJax(editor, postfix, [["$", "$"], ["\\(","\\)"]]);
      });
      });
      }, "mathjax-editing");

      StackExchange.ready(function() {
      var channelOptions = {
      tags: "".split(" "),
      id: "69"
      };
      initTagRenderer("".split(" "), "".split(" "), channelOptions);

      StackExchange.using("externalEditor", function() {
      // Have to fire editor after snippets, if snippets enabled
      if (StackExchange.settings.snippets.snippetsEnabled) {
      StackExchange.using("snippets", function() {
      createEditor();
      });
      }
      else {
      createEditor();
      }
      });

      function createEditor() {
      StackExchange.prepareEditor({
      heartbeatType: 'answer',
      autoActivateHeartbeat: false,
      convertImagesToLinks: true,
      noModals: true,
      showLowRepImageUploadWarning: true,
      reputationToPostImages: 10,
      bindNavPrevention: true,
      postfix: "",
      imageUploader: {
      brandingHtml: "Powered by u003ca class="icon-imgur-white" href="https://imgur.com/"u003eu003c/au003e",
      contentPolicyHtml: "User contributions licensed under u003ca href="https://creativecommons.org/licenses/by-sa/3.0/"u003ecc by-sa 3.0 with attribution requiredu003c/au003e u003ca href="https://stackoverflow.com/legal/content-policy"u003e(content policy)u003c/au003e",
      allowUrls: true
      },
      noCode: true, onDemand: true,
      discardSelector: ".discard-answer"
      ,immediatelyShowMarkdownHelp:true
      });


      }
      });














      draft saved

      draft discarded


















      StackExchange.ready(
      function () {
      StackExchange.openid.initPostLogin('.new-post-login', 'https%3a%2f%2fmath.stackexchange.com%2fquestions%2f3060742%2fhow-to-take-every-third-element-in-a-series%23new-answer', 'question_page');
      }
      );

      Post as a guest















      Required, but never shown

























      4 Answers
      4






      active

      oldest

      votes








      4 Answers
      4






      active

      oldest

      votes









      active

      oldest

      votes






      active

      oldest

      votes









      6












      $begingroup$

      Note that we have



      $$psi'(z)=sum_{n=0}^infty frac{1}{(n+z)^2}$$



      where $psi'(z)$ is the derivative of the digamma function. Hence, we can write



      $$sum_{n=0}^infty frac{1}{(3n+1)^2}=frac19 psi'(1/3)$$



      and



      $$sum_{n=0}^infty frac{1}{(3n+2)^2}=frac19 psi'(2/3)$$



      Interestingly, since we have



      $$sum_{n=0}^infty left(frac1{(3n+3)^2}+frac1{(3n+2)^2}+frac1{(3n+1)^2}right)=frac{pi^2}{6}$$



      we find that



      $$psi'(1/3)+psi'(2/3) = 4pi^2/3$$






      share|cite|improve this answer











      $endgroup$













      • $begingroup$
        Why does one psi lack a '
        $endgroup$
        – Dale
        Jan 13 at 18:13










      • $begingroup$
        @dale That ommission is an obvious typographical error. I've edited accordingly.
        $endgroup$
        – Mark Viola
        Jan 13 at 18:59










      • $begingroup$
        Yes, at least one of us. Note that $sum_{n=0}^infty left(frac1{(3n+3)^2}+frac1{(3n+2)^2}+frac1{(3n+1)^2}right)=frac{pi^2}{6}$ and hence $$frac19(psi'(1/3)+psi'(2/3))=frac{pi^2}{6}-frac{pi^2}{54}=frac{8pi^2}{54}=frac19left(frac{4pi^2}{3}right)$$Now multiply by $9$.
        $endgroup$
        – Mark Viola
        Jan 13 at 19:29
















      6












      $begingroup$

      Note that we have



      $$psi'(z)=sum_{n=0}^infty frac{1}{(n+z)^2}$$



      where $psi'(z)$ is the derivative of the digamma function. Hence, we can write



      $$sum_{n=0}^infty frac{1}{(3n+1)^2}=frac19 psi'(1/3)$$



      and



      $$sum_{n=0}^infty frac{1}{(3n+2)^2}=frac19 psi'(2/3)$$



      Interestingly, since we have



      $$sum_{n=0}^infty left(frac1{(3n+3)^2}+frac1{(3n+2)^2}+frac1{(3n+1)^2}right)=frac{pi^2}{6}$$



      we find that



      $$psi'(1/3)+psi'(2/3) = 4pi^2/3$$






      share|cite|improve this answer











      $endgroup$













      • $begingroup$
        Why does one psi lack a '
        $endgroup$
        – Dale
        Jan 13 at 18:13










      • $begingroup$
        @dale That ommission is an obvious typographical error. I've edited accordingly.
        $endgroup$
        – Mark Viola
        Jan 13 at 18:59










      • $begingroup$
        Yes, at least one of us. Note that $sum_{n=0}^infty left(frac1{(3n+3)^2}+frac1{(3n+2)^2}+frac1{(3n+1)^2}right)=frac{pi^2}{6}$ and hence $$frac19(psi'(1/3)+psi'(2/3))=frac{pi^2}{6}-frac{pi^2}{54}=frac{8pi^2}{54}=frac19left(frac{4pi^2}{3}right)$$Now multiply by $9$.
        $endgroup$
        – Mark Viola
        Jan 13 at 19:29














      6












      6








      6





      $begingroup$

      Note that we have



      $$psi'(z)=sum_{n=0}^infty frac{1}{(n+z)^2}$$



      where $psi'(z)$ is the derivative of the digamma function. Hence, we can write



      $$sum_{n=0}^infty frac{1}{(3n+1)^2}=frac19 psi'(1/3)$$



      and



      $$sum_{n=0}^infty frac{1}{(3n+2)^2}=frac19 psi'(2/3)$$



      Interestingly, since we have



      $$sum_{n=0}^infty left(frac1{(3n+3)^2}+frac1{(3n+2)^2}+frac1{(3n+1)^2}right)=frac{pi^2}{6}$$



      we find that



      $$psi'(1/3)+psi'(2/3) = 4pi^2/3$$






      share|cite|improve this answer











      $endgroup$



      Note that we have



      $$psi'(z)=sum_{n=0}^infty frac{1}{(n+z)^2}$$



      where $psi'(z)$ is the derivative of the digamma function. Hence, we can write



      $$sum_{n=0}^infty frac{1}{(3n+1)^2}=frac19 psi'(1/3)$$



      and



      $$sum_{n=0}^infty frac{1}{(3n+2)^2}=frac19 psi'(2/3)$$



      Interestingly, since we have



      $$sum_{n=0}^infty left(frac1{(3n+3)^2}+frac1{(3n+2)^2}+frac1{(3n+1)^2}right)=frac{pi^2}{6}$$



      we find that



      $$psi'(1/3)+psi'(2/3) = 4pi^2/3$$







      share|cite|improve this answer














      share|cite|improve this answer



      share|cite|improve this answer








      edited Jan 13 at 18:59

























      answered Jan 3 at 17:02









      Mark ViolaMark Viola

      131k1275171




      131k1275171












      • $begingroup$
        Why does one psi lack a '
        $endgroup$
        – Dale
        Jan 13 at 18:13










      • $begingroup$
        @dale That ommission is an obvious typographical error. I've edited accordingly.
        $endgroup$
        – Mark Viola
        Jan 13 at 18:59










      • $begingroup$
        Yes, at least one of us. Note that $sum_{n=0}^infty left(frac1{(3n+3)^2}+frac1{(3n+2)^2}+frac1{(3n+1)^2}right)=frac{pi^2}{6}$ and hence $$frac19(psi'(1/3)+psi'(2/3))=frac{pi^2}{6}-frac{pi^2}{54}=frac{8pi^2}{54}=frac19left(frac{4pi^2}{3}right)$$Now multiply by $9$.
        $endgroup$
        – Mark Viola
        Jan 13 at 19:29


















      • $begingroup$
        Why does one psi lack a '
        $endgroup$
        – Dale
        Jan 13 at 18:13










      • $begingroup$
        @dale That ommission is an obvious typographical error. I've edited accordingly.
        $endgroup$
        – Mark Viola
        Jan 13 at 18:59










      • $begingroup$
        Yes, at least one of us. Note that $sum_{n=0}^infty left(frac1{(3n+3)^2}+frac1{(3n+2)^2}+frac1{(3n+1)^2}right)=frac{pi^2}{6}$ and hence $$frac19(psi'(1/3)+psi'(2/3))=frac{pi^2}{6}-frac{pi^2}{54}=frac{8pi^2}{54}=frac19left(frac{4pi^2}{3}right)$$Now multiply by $9$.
        $endgroup$
        – Mark Viola
        Jan 13 at 19:29
















      $begingroup$
      Why does one psi lack a '
      $endgroup$
      – Dale
      Jan 13 at 18:13




      $begingroup$
      Why does one psi lack a '
      $endgroup$
      – Dale
      Jan 13 at 18:13












      $begingroup$
      @dale That ommission is an obvious typographical error. I've edited accordingly.
      $endgroup$
      – Mark Viola
      Jan 13 at 18:59




      $begingroup$
      @dale That ommission is an obvious typographical error. I've edited accordingly.
      $endgroup$
      – Mark Viola
      Jan 13 at 18:59












      $begingroup$
      Yes, at least one of us. Note that $sum_{n=0}^infty left(frac1{(3n+3)^2}+frac1{(3n+2)^2}+frac1{(3n+1)^2}right)=frac{pi^2}{6}$ and hence $$frac19(psi'(1/3)+psi'(2/3))=frac{pi^2}{6}-frac{pi^2}{54}=frac{8pi^2}{54}=frac19left(frac{4pi^2}{3}right)$$Now multiply by $9$.
      $endgroup$
      – Mark Viola
      Jan 13 at 19:29




      $begingroup$
      Yes, at least one of us. Note that $sum_{n=0}^infty left(frac1{(3n+3)^2}+frac1{(3n+2)^2}+frac1{(3n+1)^2}right)=frac{pi^2}{6}$ and hence $$frac19(psi'(1/3)+psi'(2/3))=frac{pi^2}{6}-frac{pi^2}{54}=frac{8pi^2}{54}=frac19left(frac{4pi^2}{3}right)$$Now multiply by $9$.
      $endgroup$
      – Mark Viola
      Jan 13 at 19:29











      3












      $begingroup$

      As a complement to Mark's answer,



      $$sum_{ngeq 0}frac{1}{(3n+1)^2}=-int_{0}^{1}sum_{ngeq 0} x^{3n}log(x),dx=int_{0}^{1}frac{-log x}{1-x^3},dx $$
      (and similarly $sum_{ngeq 0}frac{1}{(3n+2)^2}$) can be expressed in terms of dilogarithms, since
      $$ int_{0}^{1}frac{-log x}{1-a x}=frac{text{Li}_2(a)}{a} $$
      for any $|a|leq 1$, with $text{Li}_2(a)=sum_{ngeq 1}frac{a^n}{n^2}$. This is equivalent to stating that $psi'left(frac{1}{3}right)$ and $psi'left(frac{2}{3}right)$ can be computed through the discrete Fourier transform. It is worth noticing that
      $$text{Re},text{Li}_2(e^{itheta})=sum_{ngeq 1}frac{cos(ntheta)}{n^2} $$
      is a continuous and piecewise-parabolic function, as the formal primitive of the sawtooth wave. On the contrary, $text{Im},text{Li}_2(e^{itheta})$ does not have a nice closed form, in general. Ref.: https://en.wikipedia.org/wiki/Spence%27s_function






      share|cite|improve this answer









      $endgroup$


















        3












        $begingroup$

        As a complement to Mark's answer,



        $$sum_{ngeq 0}frac{1}{(3n+1)^2}=-int_{0}^{1}sum_{ngeq 0} x^{3n}log(x),dx=int_{0}^{1}frac{-log x}{1-x^3},dx $$
        (and similarly $sum_{ngeq 0}frac{1}{(3n+2)^2}$) can be expressed in terms of dilogarithms, since
        $$ int_{0}^{1}frac{-log x}{1-a x}=frac{text{Li}_2(a)}{a} $$
        for any $|a|leq 1$, with $text{Li}_2(a)=sum_{ngeq 1}frac{a^n}{n^2}$. This is equivalent to stating that $psi'left(frac{1}{3}right)$ and $psi'left(frac{2}{3}right)$ can be computed through the discrete Fourier transform. It is worth noticing that
        $$text{Re},text{Li}_2(e^{itheta})=sum_{ngeq 1}frac{cos(ntheta)}{n^2} $$
        is a continuous and piecewise-parabolic function, as the formal primitive of the sawtooth wave. On the contrary, $text{Im},text{Li}_2(e^{itheta})$ does not have a nice closed form, in general. Ref.: https://en.wikipedia.org/wiki/Spence%27s_function






        share|cite|improve this answer









        $endgroup$
















          3












          3








          3





          $begingroup$

          As a complement to Mark's answer,



          $$sum_{ngeq 0}frac{1}{(3n+1)^2}=-int_{0}^{1}sum_{ngeq 0} x^{3n}log(x),dx=int_{0}^{1}frac{-log x}{1-x^3},dx $$
          (and similarly $sum_{ngeq 0}frac{1}{(3n+2)^2}$) can be expressed in terms of dilogarithms, since
          $$ int_{0}^{1}frac{-log x}{1-a x}=frac{text{Li}_2(a)}{a} $$
          for any $|a|leq 1$, with $text{Li}_2(a)=sum_{ngeq 1}frac{a^n}{n^2}$. This is equivalent to stating that $psi'left(frac{1}{3}right)$ and $psi'left(frac{2}{3}right)$ can be computed through the discrete Fourier transform. It is worth noticing that
          $$text{Re},text{Li}_2(e^{itheta})=sum_{ngeq 1}frac{cos(ntheta)}{n^2} $$
          is a continuous and piecewise-parabolic function, as the formal primitive of the sawtooth wave. On the contrary, $text{Im},text{Li}_2(e^{itheta})$ does not have a nice closed form, in general. Ref.: https://en.wikipedia.org/wiki/Spence%27s_function






          share|cite|improve this answer









          $endgroup$



          As a complement to Mark's answer,



          $$sum_{ngeq 0}frac{1}{(3n+1)^2}=-int_{0}^{1}sum_{ngeq 0} x^{3n}log(x),dx=int_{0}^{1}frac{-log x}{1-x^3},dx $$
          (and similarly $sum_{ngeq 0}frac{1}{(3n+2)^2}$) can be expressed in terms of dilogarithms, since
          $$ int_{0}^{1}frac{-log x}{1-a x}=frac{text{Li}_2(a)}{a} $$
          for any $|a|leq 1$, with $text{Li}_2(a)=sum_{ngeq 1}frac{a^n}{n^2}$. This is equivalent to stating that $psi'left(frac{1}{3}right)$ and $psi'left(frac{2}{3}right)$ can be computed through the discrete Fourier transform. It is worth noticing that
          $$text{Re},text{Li}_2(e^{itheta})=sum_{ngeq 1}frac{cos(ntheta)}{n^2} $$
          is a continuous and piecewise-parabolic function, as the formal primitive of the sawtooth wave. On the contrary, $text{Im},text{Li}_2(e^{itheta})$ does not have a nice closed form, in general. Ref.: https://en.wikipedia.org/wiki/Spence%27s_function







          share|cite|improve this answer












          share|cite|improve this answer



          share|cite|improve this answer










          answered Jan 3 at 20:40









          Jack D'AurizioJack D'Aurizio

          288k33280659




          288k33280659























              1












              $begingroup$

              Polylogarithms



              A useful formula that can be applied here is
              $$
              frac13sum_{k=0}^2e^{2pi ijk/3}=[3mid j]tag1
              $$

              So
              $$
              begin{align}newcommand{Li}{operatorname{Li}}
              sum_{j=0}^inftyfrac1{(3j+1)^2}
              &=frac13sum_{k=0}^2sum_{j=1}^infty e^{2pi i(j-1)k/3}frac1{j^2}\
              &=frac13left(frac{pi^2}6+e^{-2pi i/3}Li_2left(e^{2pi i/3}right)+e^{2pi i/3}Li_2left(e^{-2pi i/3}right)right)tag2
              end{align}
              $$

              Mathematica gives $1.12173301393634378687$ using
              N[1/3(Pi^2/6 + Exp[-2Pi I/3]PolyLog[2,Exp[2Pi I/3]]+
              Exp[2Pi I/3] PolyLog[2,Exp[-2Pi I/3]]),20]





              Extended Harmonic Numbers



              Another approach is to use the Extended Harmonic Numbers.
              $$
              H(x)=sum_{k=1}^inftyleft(frac1k-frac1{k+x}right)tag3
              $$

              where
              $$
              H'(x)=sum_{k=1}^inftyfrac1{(k+x)^2}tag5
              $$

              Giving
              $$
              frac19H'!left(-frac23right)=sum_{k=0}^inftyfrac1{(3k+1)^2}tag6
              $$

              Mathematica gives $1.1217330139363437869$ using
              N[1/9HarmonicNumber'[-2/3],20]





              Euler-Maclaurin Sum Formula



              Although it does not give a closed form, the Euler-Maclaurin Sum Formula allows us to accelerate the computation of the sum.
              $$
              begin{align}
              sum_{k=0}^nfrac1{(3k+1)^2}
              &sim C-frac1{3(3n+1)}+frac1{2(3n+1)^2}-frac1{2(3n+1)^3}+frac9{10(3n+1)^5}\
              &-frac{81}{14(3n+1)^7}+frac{729}{10(3n+1)^9}-frac{32805}{22(3n+1)^{11}}tag7
              end{align}
              $$

              Using $n=100$ in $(7)$, we get
              $$
              sum_{k=0}^inftyfrac1{(3k+1)^2}=1.1217330139363437868657782tag8
              $$






              share|cite|improve this answer











              $endgroup$


















                1












                $begingroup$

                Polylogarithms



                A useful formula that can be applied here is
                $$
                frac13sum_{k=0}^2e^{2pi ijk/3}=[3mid j]tag1
                $$

                So
                $$
                begin{align}newcommand{Li}{operatorname{Li}}
                sum_{j=0}^inftyfrac1{(3j+1)^2}
                &=frac13sum_{k=0}^2sum_{j=1}^infty e^{2pi i(j-1)k/3}frac1{j^2}\
                &=frac13left(frac{pi^2}6+e^{-2pi i/3}Li_2left(e^{2pi i/3}right)+e^{2pi i/3}Li_2left(e^{-2pi i/3}right)right)tag2
                end{align}
                $$

                Mathematica gives $1.12173301393634378687$ using
                N[1/3(Pi^2/6 + Exp[-2Pi I/3]PolyLog[2,Exp[2Pi I/3]]+
                Exp[2Pi I/3] PolyLog[2,Exp[-2Pi I/3]]),20]





                Extended Harmonic Numbers



                Another approach is to use the Extended Harmonic Numbers.
                $$
                H(x)=sum_{k=1}^inftyleft(frac1k-frac1{k+x}right)tag3
                $$

                where
                $$
                H'(x)=sum_{k=1}^inftyfrac1{(k+x)^2}tag5
                $$

                Giving
                $$
                frac19H'!left(-frac23right)=sum_{k=0}^inftyfrac1{(3k+1)^2}tag6
                $$

                Mathematica gives $1.1217330139363437869$ using
                N[1/9HarmonicNumber'[-2/3],20]





                Euler-Maclaurin Sum Formula



                Although it does not give a closed form, the Euler-Maclaurin Sum Formula allows us to accelerate the computation of the sum.
                $$
                begin{align}
                sum_{k=0}^nfrac1{(3k+1)^2}
                &sim C-frac1{3(3n+1)}+frac1{2(3n+1)^2}-frac1{2(3n+1)^3}+frac9{10(3n+1)^5}\
                &-frac{81}{14(3n+1)^7}+frac{729}{10(3n+1)^9}-frac{32805}{22(3n+1)^{11}}tag7
                end{align}
                $$

                Using $n=100$ in $(7)$, we get
                $$
                sum_{k=0}^inftyfrac1{(3k+1)^2}=1.1217330139363437868657782tag8
                $$






                share|cite|improve this answer











                $endgroup$
















                  1












                  1








                  1





                  $begingroup$

                  Polylogarithms



                  A useful formula that can be applied here is
                  $$
                  frac13sum_{k=0}^2e^{2pi ijk/3}=[3mid j]tag1
                  $$

                  So
                  $$
                  begin{align}newcommand{Li}{operatorname{Li}}
                  sum_{j=0}^inftyfrac1{(3j+1)^2}
                  &=frac13sum_{k=0}^2sum_{j=1}^infty e^{2pi i(j-1)k/3}frac1{j^2}\
                  &=frac13left(frac{pi^2}6+e^{-2pi i/3}Li_2left(e^{2pi i/3}right)+e^{2pi i/3}Li_2left(e^{-2pi i/3}right)right)tag2
                  end{align}
                  $$

                  Mathematica gives $1.12173301393634378687$ using
                  N[1/3(Pi^2/6 + Exp[-2Pi I/3]PolyLog[2,Exp[2Pi I/3]]+
                  Exp[2Pi I/3] PolyLog[2,Exp[-2Pi I/3]]),20]





                  Extended Harmonic Numbers



                  Another approach is to use the Extended Harmonic Numbers.
                  $$
                  H(x)=sum_{k=1}^inftyleft(frac1k-frac1{k+x}right)tag3
                  $$

                  where
                  $$
                  H'(x)=sum_{k=1}^inftyfrac1{(k+x)^2}tag5
                  $$

                  Giving
                  $$
                  frac19H'!left(-frac23right)=sum_{k=0}^inftyfrac1{(3k+1)^2}tag6
                  $$

                  Mathematica gives $1.1217330139363437869$ using
                  N[1/9HarmonicNumber'[-2/3],20]





                  Euler-Maclaurin Sum Formula



                  Although it does not give a closed form, the Euler-Maclaurin Sum Formula allows us to accelerate the computation of the sum.
                  $$
                  begin{align}
                  sum_{k=0}^nfrac1{(3k+1)^2}
                  &sim C-frac1{3(3n+1)}+frac1{2(3n+1)^2}-frac1{2(3n+1)^3}+frac9{10(3n+1)^5}\
                  &-frac{81}{14(3n+1)^7}+frac{729}{10(3n+1)^9}-frac{32805}{22(3n+1)^{11}}tag7
                  end{align}
                  $$

                  Using $n=100$ in $(7)$, we get
                  $$
                  sum_{k=0}^inftyfrac1{(3k+1)^2}=1.1217330139363437868657782tag8
                  $$






                  share|cite|improve this answer











                  $endgroup$



                  Polylogarithms



                  A useful formula that can be applied here is
                  $$
                  frac13sum_{k=0}^2e^{2pi ijk/3}=[3mid j]tag1
                  $$

                  So
                  $$
                  begin{align}newcommand{Li}{operatorname{Li}}
                  sum_{j=0}^inftyfrac1{(3j+1)^2}
                  &=frac13sum_{k=0}^2sum_{j=1}^infty e^{2pi i(j-1)k/3}frac1{j^2}\
                  &=frac13left(frac{pi^2}6+e^{-2pi i/3}Li_2left(e^{2pi i/3}right)+e^{2pi i/3}Li_2left(e^{-2pi i/3}right)right)tag2
                  end{align}
                  $$

                  Mathematica gives $1.12173301393634378687$ using
                  N[1/3(Pi^2/6 + Exp[-2Pi I/3]PolyLog[2,Exp[2Pi I/3]]+
                  Exp[2Pi I/3] PolyLog[2,Exp[-2Pi I/3]]),20]





                  Extended Harmonic Numbers



                  Another approach is to use the Extended Harmonic Numbers.
                  $$
                  H(x)=sum_{k=1}^inftyleft(frac1k-frac1{k+x}right)tag3
                  $$

                  where
                  $$
                  H'(x)=sum_{k=1}^inftyfrac1{(k+x)^2}tag5
                  $$

                  Giving
                  $$
                  frac19H'!left(-frac23right)=sum_{k=0}^inftyfrac1{(3k+1)^2}tag6
                  $$

                  Mathematica gives $1.1217330139363437869$ using
                  N[1/9HarmonicNumber'[-2/3],20]





                  Euler-Maclaurin Sum Formula



                  Although it does not give a closed form, the Euler-Maclaurin Sum Formula allows us to accelerate the computation of the sum.
                  $$
                  begin{align}
                  sum_{k=0}^nfrac1{(3k+1)^2}
                  &sim C-frac1{3(3n+1)}+frac1{2(3n+1)^2}-frac1{2(3n+1)^3}+frac9{10(3n+1)^5}\
                  &-frac{81}{14(3n+1)^7}+frac{729}{10(3n+1)^9}-frac{32805}{22(3n+1)^{11}}tag7
                  end{align}
                  $$

                  Using $n=100$ in $(7)$, we get
                  $$
                  sum_{k=0}^inftyfrac1{(3k+1)^2}=1.1217330139363437868657782tag8
                  $$







                  share|cite|improve this answer














                  share|cite|improve this answer



                  share|cite|improve this answer








                  edited Jan 14 at 3:19

























                  answered Jan 13 at 20:48









                  robjohnrobjohn

                  266k27304626




                  266k27304626























                      -1












                      $begingroup$

                      KM101 deleted his hint... not sure why.



                      $frac{1}{3^2}+frac{1}{6^2}+frac{1}{9^2}+dots=frac{1}{3^2 1^2}+frac{1}{3^2 2^2}+frac{1}{3^2 3^3}+dots=frac{1}{9}(frac{1}{1^2}+frac{1}{2^2}+frac{1}{3^2}+dots)= frac{pi^2}{54}$



                      $frac{1}{2^2}+frac{1}{5^2}+frac{1}{8^2}+dots=frac{1}{2^2 1^2}+frac{1}{3^2?? 1^2}+frac{1}{3^2 ??1^2}$






                      share|cite|improve this answer











                      $endgroup$









                      • 1




                        $begingroup$
                        How does this answer the OP's question?
                        $endgroup$
                        – Mark Viola
                        Jan 3 at 20:24
















                      -1












                      $begingroup$

                      KM101 deleted his hint... not sure why.



                      $frac{1}{3^2}+frac{1}{6^2}+frac{1}{9^2}+dots=frac{1}{3^2 1^2}+frac{1}{3^2 2^2}+frac{1}{3^2 3^3}+dots=frac{1}{9}(frac{1}{1^2}+frac{1}{2^2}+frac{1}{3^2}+dots)= frac{pi^2}{54}$



                      $frac{1}{2^2}+frac{1}{5^2}+frac{1}{8^2}+dots=frac{1}{2^2 1^2}+frac{1}{3^2?? 1^2}+frac{1}{3^2 ??1^2}$






                      share|cite|improve this answer











                      $endgroup$









                      • 1




                        $begingroup$
                        How does this answer the OP's question?
                        $endgroup$
                        – Mark Viola
                        Jan 3 at 20:24














                      -1












                      -1








                      -1





                      $begingroup$

                      KM101 deleted his hint... not sure why.



                      $frac{1}{3^2}+frac{1}{6^2}+frac{1}{9^2}+dots=frac{1}{3^2 1^2}+frac{1}{3^2 2^2}+frac{1}{3^2 3^3}+dots=frac{1}{9}(frac{1}{1^2}+frac{1}{2^2}+frac{1}{3^2}+dots)= frac{pi^2}{54}$



                      $frac{1}{2^2}+frac{1}{5^2}+frac{1}{8^2}+dots=frac{1}{2^2 1^2}+frac{1}{3^2?? 1^2}+frac{1}{3^2 ??1^2}$






                      share|cite|improve this answer











                      $endgroup$



                      KM101 deleted his hint... not sure why.



                      $frac{1}{3^2}+frac{1}{6^2}+frac{1}{9^2}+dots=frac{1}{3^2 1^2}+frac{1}{3^2 2^2}+frac{1}{3^2 3^3}+dots=frac{1}{9}(frac{1}{1^2}+frac{1}{2^2}+frac{1}{3^2}+dots)= frac{pi^2}{54}$



                      $frac{1}{2^2}+frac{1}{5^2}+frac{1}{8^2}+dots=frac{1}{2^2 1^2}+frac{1}{3^2?? 1^2}+frac{1}{3^2 ??1^2}$







                      share|cite|improve this answer














                      share|cite|improve this answer



                      share|cite|improve this answer








                      edited Jan 13 at 18:15

























                      answered Jan 3 at 16:37









                      DaleDale

                      1,1801334




                      1,1801334








                      • 1




                        $begingroup$
                        How does this answer the OP's question?
                        $endgroup$
                        – Mark Viola
                        Jan 3 at 20:24














                      • 1




                        $begingroup$
                        How does this answer the OP's question?
                        $endgroup$
                        – Mark Viola
                        Jan 3 at 20:24








                      1




                      1




                      $begingroup$
                      How does this answer the OP's question?
                      $endgroup$
                      – Mark Viola
                      Jan 3 at 20:24




                      $begingroup$
                      How does this answer the OP's question?
                      $endgroup$
                      – Mark Viola
                      Jan 3 at 20:24


















                      draft saved

                      draft discarded




















































                      Thanks for contributing an answer to Mathematics Stack Exchange!


                      • Please be sure to answer the question. Provide details and share your research!

                      But avoid



                      • Asking for help, clarification, or responding to other answers.

                      • Making statements based on opinion; back them up with references or personal experience.


                      Use MathJax to format equations. MathJax reference.


                      To learn more, see our tips on writing great answers.




                      draft saved


                      draft discarded














                      StackExchange.ready(
                      function () {
                      StackExchange.openid.initPostLogin('.new-post-login', 'https%3a%2f%2fmath.stackexchange.com%2fquestions%2f3060742%2fhow-to-take-every-third-element-in-a-series%23new-answer', 'question_page');
                      }
                      );

                      Post as a guest















                      Required, but never shown





















































                      Required, but never shown














                      Required, but never shown












                      Required, but never shown







                      Required, but never shown

































                      Required, but never shown














                      Required, but never shown












                      Required, but never shown







                      Required, but never shown







                      Popular posts from this blog

                      Can a sorcerer learn a 5th-level spell early by creating spell slots using the Font of Magic feature?

                      Does disintegrating a polymorphed enemy still kill it after the 2018 errata?

                      A Topological Invariant for $pi_3(U(n))$